Quantcast
  • Register
PhysicsOverflow is a next-generation academic platform for physicists and astronomers, including a community peer review system and a postgraduate-level discussion forum analogous to MathOverflow.

Welcome to PhysicsOverflow! PhysicsOverflow is an open platform for community peer review and graduate-level Physics discussion.

Please help promote PhysicsOverflow ads elsewhere if you like it.

News

PO is now at the Physics Department of Bielefeld University!

New printer friendly PO pages!

Migration to Bielefeld University was successful!

Please vote for this year's PhysicsOverflow ads!

Please do help out in categorising submissions. Submit a paper to PhysicsOverflow!

... see more

Tools for paper authors

Submit paper
Claim Paper Authorship

Tools for SE users

Search User
Reclaim SE Account
Request Account Merger
Nativise imported posts
Claim post (deleted users)
Import SE post

Users whose questions have been imported from Physics Stack Exchange, Theoretical Physics Stack Exchange, or any other Stack Exchange site are kindly requested to reclaim their account and not to register as a new user.

Public \(\beta\) tools

Report a bug with a feature
Request a new functionality
404 page design
Send feedback

Attributions

(propose a free ad)

Site Statistics

205 submissions , 163 unreviewed
5,047 questions , 2,200 unanswered
5,345 answers , 22,709 comments
1,470 users with positive rep
816 active unimported users
More ...

  Meaning of $\int \phi^\dagger \hat A \psi \:\mathrm dx$

+ 5 like - 0 dislike
2654 views

While analysing a problem in quantum Mechanics, I realized that I don't fully understand the physical meanings of certain integrals. I have been interpreting:

  • $\int \phi^\dagger \hat A \psi \:\mathrm dx$ as "(square root of) probability that a particle with state $|\psi\rangle$ will collapse to a state $|\phi\rangle$ when one tries to observe the observable corresponding to $\hat A$"

  • $\int \phi^\dagger \psi \:\mathrm dx$ as "(square root of) probability that a particle with state $|\psi\rangle$ will collapse to a state $|\phi\rangle$".

All integrals are over all space here, and all $\phi$s and $\psi$s are normalized.

Now, I realize that interpretation of the first integral doesn't really make sense when put side by side with the second one.

For example, when $|\phi\rangle$ is an eigenstate of $\hat A$, I get two different expressions for "the square root of probability that one will get the corresponding eigenvalue of $\psi$ when one tries to observe the observable corresponding to $\hat A$". As far as I can tell, $\int \phi^\dagger \hat A \psi \:\mathrm dx \neq\int \phi^\dagger \psi \:\mathrm dx $, even if $|\phi\rangle$ is an eigenstate of $\hat A$. I am inclined to believe that the second integral is the correct answer here (it comes naturally when you split $|\psi\rangle$ into a linear combination of basis vectors). But I am at a loss as to the interpretation of the first integral.

So, my question is, what are the physical interpretations of $\int \phi^\dagger \hat A \psi \:\mathrm dx$ and $\int \phi^\dagger \psi \:\mathrm dx$?

(While I am familiar with the bra-ket notation, I have not used it in this question as I don't want to confuse myself further. Feel free to use it in your answer, though)

This post imported from StackExchange Physics at 2014-06-14 13:07 (UCT), posted by SE-user Manishearth
asked Jan 27, 2013 in Astronomy by Manishearth (35 points) [ no revision ]
Your interpretation of the second integral is the most correct. That is, the second integral is like a dot product between two vectors. It represents "how much" of the amplitude of $\psi$ is in $\phi$. The first integral isn't a probability.

This post imported from StackExchange Physics at 2014-06-14 13:07 (UCT), posted by SE-user Mew
You're close to the Fermi Golden Rule en.wikipedia.org/wiki/Fermi's_golden_rule

This post imported from StackExchange Physics at 2014-06-14 13:07 (UCT), posted by SE-user Jorge
@chris thought so, since I could derive it easily by splitting $|\psi\rangle$ into a sum of basis eigenstates. Maybe you could explain what the first integral is in an answer? :)

This post imported from StackExchange Physics at 2014-06-14 13:07 (UCT), posted by SE-user Manishearth
I wish I could. I can't think of a simple interpretation of the first integral. I agree with Ondrej though that it is the overlap of $\phi$ and $A\psi$, but that's about it.

This post imported from StackExchange Physics at 2014-06-14 13:07 (UCT), posted by SE-user Mew

3 Answers

+ 7 like - 0 dislike

I'll start with the second one. $\int\phi^\ast\psi\,\mathrm{d}x$ is, as Chris says in the comments, the scalar (or dot) product of $\phi$ and $\psi$. In the Dirac notation, it is written as $\langle\phi|\psi\rangle$ and it gives the overlap of the two wavefunctions. In other words, it gives the probability amplitude (i.e., what you call square root of probability) that starting from $|\psi\rangle$ we will measure the state to be $|\phi\rangle$.

The first integral works in pretty much the same way; it's just the scalar product of $|\phi\rangle$ with $\hat{A}|\psi\rangle$. This means that you first apply $\hat{A}$ to the state $|\psi\rangle$ and then measure its overlap with $|\phi\rangle$. But this is not the same as calculating the amplitude probability of measuring $|\phi\rangle$ after the measurement of $\hat{A}$ is performed on $|\psi\rangle$. If you measured $\hat{A}$, that would be described by a set of projectors $\hat{\Pi}_i = |\chi_i\rangle\langle\chi_i|$, where $|\chi_i\rangle$ is an eigenvector of $\hat{A}$. The state $|\psi\rangle$ collapses to $|\chi_i\rangle$ with probability $p_i = |\langle\chi_i|\psi\rangle|^2$ which then collapses to $|\phi\rangle$ with probability $q_i = |\langle\phi|\chi_i\rangle|^2$. The overall probability is then $P = \sum_i p_i q_i$, which is not the same as $|\langle\phi|\hat{A}|\psi\rangle|^2$, as you can see if you write $\hat{A}|\psi\rangle = \sum_i \chi_i|\chi_i\rangle\langle\chi_i|\psi\rangle$, with $\chi_i$ being the eigenvalues of $\hat{A}$.

Edit:

The interpretation of $\hat{A}|\psi\rangle$ can be divided into several cases:

  1. If $\hat{A}$ is unitary, $\hat{A}|\psi\rangle$ can be understood as a time evolution of $|\psi\rangle$ governed by a Hamiltonian $\hat{H}$ fulfilling $\hat{A} = \exp(i\hat{H}t)$.

  2. If $\hat{A}$ is not unitary, but is trace-decreasing (i.e., $\langle\psi|\hat{A}^\dagger\hat{A}|\psi\rangle \le \langle\psi|\psi\rangle$ for each $|\psi\rangle$) it can describe some probabilistic evolution $|\psi\rangle\to|\varphi_i\rangle$ with probability $p_i\le 1$.

  3. The general case cannot be, I believe, interpreted generally. It requires renormalization (as does case 2) but this time the norm cannot be interpreted as the probability. Thus, it depends on the specific operator used.

As daaxix pointed out in the comments, the expression $\langle\phi|\hat{A}|\psi\rangle$ can be understood as a form of generalized mean value. Assuming $\hat{A}$ is Hermitian and writing $|\psi\rangle = \sum_k c_k|\chi_k\rangle$, $|\phi\rangle = \sum_k d_k|\chi_k\rangle$, we have $$ \langle\phi|\hat{A}|\psi\rangle = \sum_{k,l} c_k d_l^\ast \langle\chi_l|\hat{A}|\chi_k\rangle.$$ Due to orthogonality of $|\chi_k\rangle$, we can write $\langle\chi_l|\hat{A}|\chi_k\rangle = \delta_{kl} \langle\chi_k|\hat{A}|\chi_k\rangle$, so we have $$ \langle\phi|\hat{A}|\psi\rangle = \sum_k c_k d_k^\ast \langle\chi_k|\hat{A}|\chi_k\rangle. $$ This gives a sum of the expectation values $\langle\chi_k|\hat{A}|\chi_k\rangle$ (i.e., the eigenvalues $\chi_k$), with weight coefficients given by the expansion of the vectors $|\psi\rangle$, $|\phi\rangle$, namely $c_k d_k^\ast$.

This post imported from StackExchange Physics at 2014-06-14 13:07 (UCT), posted by SE-user Ondřej Černotík
answered Jan 27, 2013 by Ondřej Černotík (70 points) [ no revision ]
Thanks for your answer (I'll accept tomorrow after giving others a chance to answer). Is there any way to interpret $\hat A|\psi\rangle$? Even a slightly mathematical/complicated explanation would do (editing it into your answer would be nice)

This post imported from StackExchange Physics at 2014-06-14 13:08 (UCT), posted by SE-user Manishearth
Not a completely general one, as far as I know. I'll try and add something more on the topic tomorrow, don't have much time now...

This post imported from StackExchange Physics at 2014-06-14 13:08 (UCT), posted by SE-user Ondřej Černotík
@Ondejernotík: "Not a completely general one" -- that's intriguing, my interest is piqued :) No problem, add it whenever you have the time..

This post imported from StackExchange Physics at 2014-06-14 13:08 (UCT), posted by SE-user Manishearth
It's not a big deal, there are just several possibilities and I think it's better to treat them separately..

This post imported from StackExchange Physics at 2014-06-14 13:08 (UCT), posted by SE-user Ondřej Černotík
The interpretation of the first integral is exactly the sum $\sum C_k\langle \hat{A}\rangle$, or the sum of expectation values over a canonical orthogonal wavefunction basis, where both $\phi$ and $\psi$ can be represented in that orthogonal basis. So it is sort of like a weighted expectation value, weighted by the respective wavefunctions $\phi$ and $\psi$.

This post imported from StackExchange Physics at 2014-06-14 13:08 (UCT), posted by SE-user daaxix
@daaxix Thanks for the observation, I'll edit it to the answer :)

This post imported from StackExchange Physics at 2014-06-14 13:08 (UCT), posted by SE-user Ondřej Černotík
Great edit, the whole concept makes a lot more sense to me now. I like the "weighted expectation value" interpretation :)

This post imported from StackExchange Physics at 2014-06-14 13:08 (UCT), posted by SE-user Manishearth
@Manishearth Glad it helped :)

This post imported from StackExchange Physics at 2014-06-14 13:08 (UCT), posted by SE-user Ondřej Černotík
+ 2 like - 0 dislike

Think about vectors in $\mathbb{R}^2$. The inner product over this space gives you a notion of an angle and orthogonality. For example $\hat x \cdot \hat y= 0$ means they are completely orthogonal and you cannot express one in terms of the other.

This is the same interpretation for $\langle f | g\rangle = \displaystyle \int_a ^b f^*g \ dx$ which is an inner product over the space $L^2(a,b)$ where the solutions to the Schrodinger operator live. Same interpretations for $ z=x \cdot A y$ where $x,y \in \mathbb{R}^2$ and $A \in M_2(\mathbb{R})$ apply for $\langle f |A| g\rangle$ with $A\in \mathcal{L}(L^2)$ a linear operator over $L^2$.

This post imported from StackExchange Physics at 2014-06-14 13:08 (UCT), posted by SE-user yca
answered Jan 27, 2013 by yca (20 points) [ no revision ]
You answer is basically the same as Ondrej. So what is the meaning of $A|g>$? It is clearly some kind of rotation. But after that why do you want to project it onto the $<f|$, what is the meaning then?

This post imported from StackExchange Physics at 2014-06-14 13:08 (UCT), posted by SE-user hwlau
it need not be a "rotation". if by "meaning" you refer to the operator's counterpart in nature, well that will depend on the physical problem you are working on.

This post imported from StackExchange Physics at 2014-06-14 13:08 (UCT), posted by SE-user yca
+ 2 like - 0 dislike

The mathematical operation of applying $\hat{A}$ to a ket $| \phi \rangle$, is a generalized rotation in Hilbert space, that results in another ket $\hat{A}| \phi \rangle$ which may be useful in further calculations, but is not in general the result of measuring the physical quantity $A$.

Therefore, the quantity $\langle \psi |A| \phi \rangle$ has no simple, general interpretation. However, in many concrete problems, a quantity with physical meaning appears in that form, specially when dealing with continuous states. For example, in particle scattering, there is the scattering matrix $S$. The quantity $\langle \beta |S| \alpha \rangle$ represents the amplitude for the free-particle state $| \beta \rangle$ to be found from the initial state $| \alpha \rangle$ after the scattering has taken place. This is used to compute scattering cross sections.

Another example is that of the atomic transitions that account for the spectral lines. The different mechanisms involved (being the electric dipole the most usual) are represented by operators whose matrix elements $\langle k | E | n \rangle$ are directly related to the transition probabilities.

(Apart from that, you surely know that the special case $\langle \psi |A| \psi \rangle$ is the expectation value for the operator $\hat{A}$ for the state represented by $| \psi \rangle$, except by a normalization factor $\langle \psi |\psi \rangle$)

As for the other integral $\langle \phi | \psi \rangle$, your interpretation is essentially correct, except that it is not the square root of a probability, but rather a complex number (the difference matters when you have to add two amplitudes, for example)

This post imported from StackExchange Physics at 2014-06-14 13:08 (UCT), posted by SE-user Eduardo Guerras Valera
answered Jan 27, 2013 by Eduardo Guerras (435 points) [ no revision ]

Your answer

Please use answers only to (at least partly) answer questions. To comment, discuss, or ask for clarification, leave a comment instead.
To mask links under text, please type your text, highlight it, and click the "link" button. You can then enter your link URL.
Please consult the FAQ for as to how to format your post.
This is the answer box; if you want to write a comment instead, please use the 'add comment' button.
Live preview (may slow down editor)   Preview
Your name to display (optional):
Privacy: Your email address will only be used for sending these notifications.
Anti-spam verification:
If you are a human please identify the position of the character covered by the symbol $\varnothing$ in the following word:
p$\hbar$ysics$\varnothing$verflow
Then drag the red bullet below over the corresponding character of our banner. When you drop it there, the bullet changes to green (on slow internet connections after a few seconds).
Please complete the anti-spam verification




user contributions licensed under cc by-sa 3.0 with attribution required

Your rights
...